Download as pdf or txt
Download as pdf or txt
You are on page 1of 203

ACCESSORIES IN INTERIOR DESIGNING

B.Sc. IDD - VI SEMESTER


Introduction to Creative Art and Craft
Art, Craft and Design are three inter-dependent disciplines.

• Art emphasizes ideas, feelings, and visual qualities.

• Craft emphasizes the right use of tools and materials.

• Design emphasizes planning, problem-solving and completion, using drawing as a

means of thinking.
ART

• Art is form of creative expression. It can be verbal, written, enactment, visual or


any other form that has potential and ability to communicate thoughts, ideas and
information.
Classification Of Art

Some of the classification of art includes,


• Fine art
• Visual art
• Performance art
• Plastic art
• Applied art
• Decorative art
• Culinary art
Fine Art

• This category includes works of art that are created primarily for aesthetic
reasons.
• Fine arts include:
• Drawing – charcoal, chalk, crayon, pastel, graphite, colored pencil, or pen and
ink
• Painting – oils, watercolor, gouache, acrylics, ink and wash, tempera, or
encaustic paints
• Printmaking – woodcuts, stencils, engraving, etching and lithography, or
screen-printing, foil imaging, or giclee prints
• Sculpture – bronze, stone, marble, wood, or clay
• Calligraphy – beautiful and stylized handwriting
Visual Art

• The visual arts include all the fine arts, in addition to the following:
• New media – digital art, computer graphics, computer animation, virtual art,
Internet art, interactive art, video games, computer robotics, 3D printing, and art
as biotechnology
• Photography
• Environmental art
• Contemporary forms of expression – assemblage, collage, conceptual,
installations
Plastic Art
• The term plastic art includes artworks that are molded and not necessarily plastic
objects.
• This category consists of three-dimensional works like clay, plaster, stone, metals,
wood, and paper (origami).

Performance Art
• This classification consists of an art form that refers to public performance events.
• Performance arts include: Theatre/ drama, Dance, Music, puppetry.
Applied Art

• This category encompasses the application of aesthetic designs to everyday


functional objects.
• Applied arts are intended for the use of a career.
• It includes architecture, computer art, photography, industrial design, graphic
design, fashion design, and interior design.
Decorative Art

• This classification refers to functional but ornamental art forms.


• Such as jewelry, ceramics, mosaic art, and other embellished items by ornaments
and other designs.
• It also includes works in glass, stained glass, clay, wood, metal, textile/fabric,
furniture, furnishings, and tapestry art.
• Interior designers often use this art form for home, commercial and retail
outlets, and office decor.
Culinary Art

• Culinary arts is a broad term that refers to the preparation, cooking, plating,
presentation, and service of food.
Craft

• Indian words for handicrafts are commonly hastkala, hastshilp, dastkari, karigari,
all meaning handiwork, but they also refer to objects made with craftsmanship,
i.e., specialised skills of the hands which are also artistic.
• The aesthetic content is an intrinsic part of such objects and means the object of
utility has a value that goes beyond mere usage and is also pleasing to the eye.
• A handcrafted object is seldom merely decorative, and whether it has no
embellishment or is highly decorative, its true purpose is served only when it is
both useful and has a fine form.
• Crafts are closely related to concept of form, pattern, design, usage, and these
lead to its total aesthetic quality.
Crafts and Culture

• Craft is rooted in the culture of the people in a particular area of a country or


among certain communities, crafts become a part of its cultural heritage.
• Handcrafted objects are not just valuable for their aesthetic quality, but as
objects produced by traditional craftsmen and women for ceremonial and
religious purposes, and most importantly as a means of livelihood.
• The creative spirit in the midst of the struggle for survival is one of the unique
and significant distinguishing features between men and beasts. Why else do the
forest communities, even today, lay so much store by painting the inside or
outside of their homes, or adorning their bodies with decorative tattoos or
ornaments? Why are people affected, and often spiritually guided, by colours?
Why does a woman fashion an attractive handle for the broom she uses to clean
her home and why does she spend time invoking the blessings of the gods
through her painted designs on her kitchen floor?
Types of Crafts

• In present-day India we can formulate broad groupings of major practitioners of


craft as:
• potters, weavers, metal-smiths, wood-carvers, cane and bamboo weavers and
stone carvers. While these may be the larger skills,
• There are many other crafts from shola pith work, paper machie, innumerable
styles of murals, masks, miniature and floor painting, paper crafts, glass work,
carpet and duree weaving.
• In the area of textiles India undoubtedly has the largest range of skills that can be
found anywhere in the world.
• While separating the craft of making floor coverings out of rags and yarn,
we are left with textiles that consist of ornamentation through pre-loom
processes, plain weaving, patterned weaving in which the ornamentation
appears during the weaving process, and post-loom ornamentation.
• The last of these can be further subdivided into embroidery, beadwork,
block printing and tie-and dye techniques, and zari (metallic thread) work,
offering a further array of skills that express themselves differently from
one region to another
Indian Folk Art
Warli painting
• Warli painting is a tribal art which comes from Maharashtra in Western India.
Traditionally it was created using a mixture of ground rice and water painted onto
the mud walls of buildings. There is no striking colour palette either.
• Simplicity is the key in Warli.
• The Warli culture is centered on the concept of Mother Nature and elements of
nature are often focal points depicted in Warli painting.
• Warli is about day-to-day life and the human connection. A vital part of it is what
one does on a regular day — agriculture, weddings,festivals, anything a common
man will do. There are no deities involved.
• Figures are created out of geometrical shapes — two triangles, stick-like hands
and legs, a circle (representing face), a smaller circle (only for female form to
indicate a hair bun) and square.
Madhubani Art
• Madhubani painting has its origins in Maithili village of Bihar.

• In the beginning, the women of the village drew these paintings on the walls of their
dwelling, as a demonstration of their feelings, hopes, and ideas.

• Later these paintings started becoming a part of festivities and special occasions.
Gradually, the Madhubani painting traversed the traditional boundaries and reached
professional artists both in India and abroad.

• Base and style - Although initially, mud walls were the canvases for these paintings, now,
cloth is used, apart from handmade paper. The themes are not diverse since the original
paintings were restricted to a particular geography. The paintings also use 3D images.

• Color - The colors are derived mostly from plants.


Themes of Madhubani Paintings

• The themes of these paintings are chiefly nature and mythological events. It revolves
around Hindu deities like Krishna, Shiva, Lakshmi, Durga, Rama, and Saraswati.

• The nature-related themes revolve around the Sun, the Moon, and sacred plants such as
basil. There are some paintings with themes based on court scenes and social events.

• If any unfilled space is left after painting the central theme, it is filled up with the motifs
of animals, birds, flowers or geometric patterns.
Making Madhubani Paintings
• Madhubani paintings use brushes created out of cotton, draped over a bamboo
stick.
• There is no shading in the application of colors.
• A double line is drawn for outlines and the gap is filled with either cross or
straight miniature lines.
• Black color is made by mixing soot to cow dung
• Yellow color from combining turmeric with the milk of banyan leaves
• Blue color from indigo
• Red colour from red sandalwood or the juice of Kusam flower
• Green color from the wood apple tree leaves
• White colour is obtained from rice powder
• Orange color from palasha flowers
Bhil Art
• Bhil paintings are artistic and imaginative art of Bhil tribe which is considered oldest art
form of tribal art of India.

• The distinctive style of Bhil painting is in-filling the motives with multicolor dots in
signature pattern (not random).

• Their paintings represent similarity with pointillism art .


• Traditionally Bhil paintings are done to decorate their houses and temples. Walls are
prepared every year by applying plaster of earth known as “mittichitra”, over which the
painting is done with natural colors.
• The tradition of wall painting survives on postcards, canvas paper and paintings in the
exhibitions.
• The Bhils follow mainly agricultural life. A strong influence of natural environment
is seen in the Bhil art.
• The basic element of Bhil art is Story-telling and artists describe various stories on
a particular ceremony through their art.
• Elements like folklores, rituals, tattoos, songs are the center point of their art.
• Their paintings also indicate the changing seasons, landscape of the fields during
harvesting and rituals of gods.
• Traditionally in Bhil art, large but un-life like shapes of common characters are
drawn. These motifs are then filled with earthy but bright colors in plain form.
• On these drawings several patterns are created with the uniform dots of multi
colors which stand out prominently against the background.
• The colors are prepared from leaves and flowers, ash, turmeric, rock or clay
which they obtained from their surrounding area and fields. To form a
paste these raw materials are crushed into powder then mixed in warm
water. Often rice powder paste with water is used as paint base.
• Neem twigs are frayed and used as brushes to fill the colors. Also to draw
some ethnic drawings traditionally middle or ring finger of right hand is
used to fill the color.
• Nowadays, for preparing traditional and modern paintings, acrylic or
synthetic colors are also used by members of the younger generations of
the Bhil tribe. Pencil and scales are also recently introduced to draw the
outline.
Kalamkari Art

• Kalamkari originated in the modern-day states of Andhra Pradesh and Telangana


several hundred years ago.
• Deriving its name from the word ‘kalam,’ which means pen, ‘Kalamkari’ refers to a
particular, intricate style of hand-painting onto cloth.
• It was first used to portray scenes from sacred texts such as the Mahabharata,
Ramayana and Bhagavatam. These paintings were often displayed as decorative
backdrops in temples, depicting the stories of deities.
• There are two types of Kalamkari painting: Srikalahasti, which is the freehand
drawing style, and Machilipatnam, which is the block-printing technique.
• The Tree of Life is one especially popular Kalamkari motif — deeply rooted while
growing towards the sky, it connects the heavens, earth and underworld. It is also
a symbol of nourishment, with many animals feeding on its leaves, living in its
branches and enjoying its shade. Peacocks, tigers and deer also frequently
appear.
• Like many traditional Indian crafts, Kalamkari uses only natural materials. The
dyes are obtained by extracting colours form parts of plants - roots, dried unripe
fruits and leaves along with mineral salts of iron, tin, copper, alum, etc., which are
used as mordants.
• The process of creating a Kalamkari painting is as elaborate as the delicate and detailed
designs — there are 23 meticulous steps. The first step is preparing the cotton to absorb
dyes by washing it to remove starch, sun-drying it fully and then treating it with mordant
as a fixative; preparing and mixing natural pigments; applying each color individually; and
rinsing the fabric between every color application.
• After tracing the central figure with charcoal sticks, the artist uses a finely pointed kalam,
or a bamboo pen, dipped in the black kasimi liquid. The kalam is wrapped in wool that
holds the liquid, and the artist squeezes this wool to release the ink while painting.
• After the black outline has dried, the artist can apply a mordant with alum and begin
introducing red color to the cloth.
• The final colors are earthy tones of reds, blues, greens, yellows and browns
Pattachitra Art
• Pattachitra art is a traditional painting style that has its roots in Odisha, India.
Etymologically speaking, Pattachitra means picture or painting on cloth, paper,
and palm leaves which is how this art form is practiced.
• This 3000-year-old art form is also practised on paper, and the artists spend
several hours creating the hand-painting depicting songs, stories, and folklore
from Odisha.
• Pattachitra Art is usually centered around the themes of the Jagannath and the
Vaishnava sect and the various avatars of Lord Krishna. Besides narrating the
stories of Lord Krishna and his incarnations, it also has flowers, foliages, and other
elements from nature on the canvas.
• Pattachitra paintings were traditionally drawn by the Mahapatra or maharanas,
the original artiste caste in Odisha.
Pattachitra Art
• Ganjapa is a Pattachitra painting which captures the social themes.
• Yamapati and yatripatas include paintings of the Puri temple.
• Saiva, Ragachitras, Shakta, and Navagunjar focus on the avtars of Lord Krishna.
• The artists who create the Pattachitra Art work as a family where the men or the
primary artist create the outline of the painting on small strips of cotton cloth.
Whereas the women work on preparing the glue, the canvas, and filling the
paintings with colours. To give the canvas a leathery finish, they rub it with a
mixture of chalk and tamarind seeds.
• The Pattachitra when painted on cloth follows a traditional process of preparation
of the canvas. First the base is prepared by coating the cloth with the soft, white,
stone powder of chalk and glue made from tamarind seeds. This gives the cloth
tensile strength and a smooth, semi-absorbent surface, allowing it to accept the
paint.
• The artist does not use a pencil or charcoal for the preliminary drawings. It is a
tradition to complete the borders of the painting first. The painter then starts
making a rough sketch directly with the brush using light red and yellow.
• The main flat colors are applied next; the colors used are normally white, red,
yellow, and black. The painter then finishes the painting with fine stokes of black
brush lines, giving the effect of pen work. When the painting is completed it is
held over a charcoal fire and lacquer is applied to the surface. This makes the
painting water resistant and durable, besides giving it a shining finish.
• The materials used in the paint are from vegetable, earth, and
mineral sources. Black is made out of lampblack, yellow from haritala
stone, and red from shingle stone. White is prepared from crushed,
boiled, and filtered shells.
• The Pattachitra artists also paint their themes on wooden boxes, on
bowls, on tussar silk, on outer shells of the coconut, and on wooden
doors. They are also working on producing painted wooden toys
based on animals and birds portrayed in the paintings.
• The artists have also traditionally painted playing cards or Ganjifa.
Chitra-pothies --- a collection of painted palm leaves stacked on top of
each other and held together between painted wood covers by
means of a string --- illustrate mythological themes.
Gond Art

• The word ‘Gond’ comes from the Dravidian expression ‘Kond’ which means ‘green
mountain’.
• Gond painting is a famous folk art of the Gond tribal community of central India.
• It is a form of painting from folk and tribal art that is practiced by one of the
largest tribes in India – the Gond – who are predominantly from Madhya Pradesh,
but can also be found in pockets of Andhra Pradesh, Maharashtra, Chhattisgarh,
and Odisha.
• Amongst the Gond people, painting and other forms of expressive art are a
popular and traditional practice. Dating back to more than 1400 years ago, the
Gond tribe has painted and carved art on the walls of caves which all belong to
the Mesolithic Period.
• The Gonds believe that viewing good images brought in good luck. Thus,
traditionally, they painted motifs, tattoos, and images on the floors and walls of
their houses. Their paintings are also a means to record and pass on historical
knowledge and teachings.
• The Pardhan Gonds are extremely skilled artists who are renowned for their
paintings and music.
• The Gond people believe that every element of nature, be it the hills, the sky or
the trees, is inhabited by a spirit. Recreating the forms of nature therefore is done
with reverence and worship of their sacred essence.
• They also depict scenes from the contemporary life of the people. A detailed
narrative portrays abstract concepts encompassing emotions, dreams, and
imaginative constructs.
• Since many of the Pardhan Gonds are accomplished musicians, their paintings
often portray the stories of their songs.
• The most significant subjects explored and depicted in Gond paintings include
birds, animals, and mythical beasts.
• Since the paintings also derive themes from the daily lives of the people, the
Mahua Tree is an important subject. Considered as the Tree of Life, the flowers,
fruits, seeds, and leaves from the Mahua tree serve many purposes for the tribe.
• Urban themes depict modern subjects such as technology and automobiles.
Folktales and local deities like Phulvari Devi, Jalharin Devi, and Marahi Devi are of
cultural importance.
• The style that is incorporated takes care that the lines, whether of the border or
the interior intricacies, that create the subjects grab the attention of the viewer
instantly.
• The dots and dashes enhance the details with exquisite brilliance that is unique
to the Gond paintings.
• Incorporating various geometric shapes and patterns, like the ones resembling
fish scales, drops of water and seed shapes flesh out the expressive value of the
art.
• The sense of movement is established through the curves and strokes of the
lines, and hence many critics best described it as ‘on line work’.
• The Gond paintings flourished with vivid colours, especially red, blue, orange,
yellow, and white, that serve the view with their excellent contrast.
• These bright paints are usually derived and extracted from organic sources such
as coloured soil, charcoal, plant leaves and sap, and even cow dung.
• The local sand called Chui Mitti helps in the production of the yellow colour, while
Gheru Mitti serves the brown colour.
• Charcoal provides the black colour, Hibiscus flower gives red, and plant leaves
impart green.
• In the modern age, one can commonly find Gond art on wooden trays, boxes, and
other objects of smooth surfaces.
• For the sake of convenience, the artists now prefer canvas. Today, Gond artists
use poster colours and other artificial paints to put their talent into the best
portrayal.
Indian Art Appreciation Study
Prehistoric Rock Paintings

• The distant past when there was no paper or language or the written word, and
hence no books or written documents, is called prehistory, or, as we often say,
prehistoric times.
• Excavation at these places brought to light old tools, pottery, habitats, bones of
ancient human beings and animals, and drawings on cave walls.
• When the basic needs of food, water, clothing and shelter were fulfilled people
felt the need to express themselves. Painting and drawing were the oldest art
forms practiced by human beings to express themselves, using the cave walls as
their canvas.
• The prehistoric rock paintings provide valuable insights into the lives and culture
of early human beings during the Old Stone Age or Palaeolithic period. These
paintings were created in the distant past, when there was no written language
or books. Scholars have pieced together information about prehistoric times by
excavating sites and studying the artifacts and cave drawings found there.
• The earliest known rock paintings were discovered in India in 1867-68, predating
the famous Altamira cave paintings in Spain. Rock paintings have been found in
various parts of the world, but the most significant sites are located in India. One
of the prominent sites is Bhimbetka, located in the Vindhya hills of Madhya
Pradesh, which contains around 800 rock shelters with paintings.
• The paintings at Bhimbetka and other sites depict a wide range of subjects, including
animals, human figures, hunting scenes, daily life activities, and sacred or royal images.
The paintings are categorized into different historical periods, such as Upper Palaeolithic,
Mesolithic, and Chalcolithic. The styles and techniques vary, with linear representations
and geometric patterns prevalent in the Upper Palaeolithic phase, while smaller hunting
scenes dominate the Mesolithic period.
• The artists used various colors, including white, red ochre, green, and black, obtained
from grinding rocks and minerals. Despite the limitations of working conditions and
materials, the paintings exhibit a pictorial quality, portraying the environment,
adventures, and struggles for survival. Animal figures are more common than human
figures, showcasing the relationship between humans and the animal kingdom.
• The prehistoric rock paintings serve as a window into the lives of early human
beings, their hunting practices, communal activities, and artistic expressions.
They also provide valuable evidence of the evolution of human civilization
through the artifacts and remains associated with the paintings. The rock
paintings, with their enduring colors, continue to captivate and reveal the artistic
sensibilities of our ancient ancestors.
Cave painting from Upper Palaeolithic period Cave painting from Mesolithic period
Hunting scene Dancing scene
Hunting scenes predominate in Mesolithic paintings. This In this picture hand-linked figures in dancing mode are
is one such scene where a group of people are shown shown. In fact, this is a recurrent theme. It also recalls the
hunting a bison. Some injured men are depicted lying dancing scene from the Lakhudiyar rock painting found in
scattered on the ground. These paintings show mastery in Uttarakhand.
the skill of drawing these forms
QnA
1. How did the people of prehistoric times select themes for their paintings?
The people of prehistoric times selected themes for their paintings based on their daily life
experiences and their surroundings. They painted scenes depicting hunting, dancing,
music, horse and elephant riders, animal fighting, honey collection, decoration of bodies,
and other household activities. These themes reflected their interactions with the natural
world and their efforts to express their experiences and emotions through art.
2. What could have been the reasons for depicting more animal figures than human figures
in cave paintings?
There could be several reasons for depicting more animal figures than human figures in
cave paintings. One reason is that animals played a crucial role in the lives of prehistoric
people. They depended on hunting animals for food, clothing, and other resources, so
depicting animals in their art might have held significant cultural and symbolic meaning.
Animals may have been seen as powerful and respected creatures, and their
representations in cave paintings could have been a way to honor and connect with them.
Additionally, animals might have held spiritual or mythological significance for prehistoric
societies, and the paintings could have served as a form of ritual or religious expression.
4. Other than Bhimbetka, which are the other major sites where these prehistoric paintings have
been found?
Besides Bhimbetka, other major sites where prehistoric paintings have been found include various
districts in Madhya Pradesh, Uttar Pradesh, Andhra Pradesh, Karnataka, Bihar, and Uttarakhand in
India. Some notable sites mentioned in the text are the Kumaon hills in Uttarakhand, the granite
rocks of Karnataka and Andhra Pradesh (sites like Kupgallu, Piklihal, and Tekkalkota), and the
Vindhya ranges of Madhya Pradesh and their Kaimurean extensions into Uttar Pradesh (including
Bhimbetka). These sites have revealed a wealth of prehistoric rock paintings, each with its unique
characteristics, artistic styles, and cultural significance.
The Arts Of The Indus Valley
• The arts of the Indus Valley Civilization emerged during the second half of the third
millennium BCE. The art forms found from the sites of civilization include sculptures,
seals, pottery, gold jeweler, terracotta figures, etc.
• The artists of that time had fine artistic sensibilities and a vivid imagination. Their
delineation of human and animal figures was highly realistic in nature, since the
anatomical details included in them was unique, and, in the case of terracotta art, the
modelling of animal figures was done in an extremely careful manner.
• The two major sites of the Indus Valley Civilization, along the Indus river—the cities of
Harappa in the north and Mohenjodaro in the south—showcase one of earliest examples
of civic planning. Houses, markets, storage facilities, offices, public baths, etc., arranged
in a grid-like pattern. There was also a highly developed drainage system.
• While Harappa and Mohenjodaro are situated in Pakistan, the important sites excavated
in India are Lothal and Dholavira in Gujarat, Rakhigarhi in Haryana, Ropar in the Punjab,
Kalibangan and Balathal in Rajasthan, etc.
Stone Statues

• The stone statues discovered at Harappa and Mohenjodaro showcase impressive


mastery of three-dimensional forms.
• Notably, there are two noteworthy male figures—a red sandstone torso and a
steatite bust of a bearded man—that have garnered extensive attention.
• The bearded man, believed to be a priest, is depicted draped in a shawl adorned
with trefoil patterns. His eyes are slightly elongated, half-closed in a meditative
state. The facial features include a well-formed nose, an average-sized mouth
with a close-cut mustache and short beard, and double shell-like ears with a
central hole. The hair is parted in the middle, and a simple woven band encircles
the head. Additionally, he wears an armlet on his right hand, and indications of a
necklace are visible through the presence of holes around his neck.
In this red sandstone
figure, there are
socket holes in the
neck and shoulders
for the attachment of
head and arms. The
frontal posture of the
torso has been
consciously adopted.
The shoulders are
well baked and the
Male Torso abdomen slightly
prominent.
Bust of a bearded priest
Bronze Casting

• The Harappans extensively practiced the art of bronze casting using the 'lost wax'
technique.
• This involved coating wax figures with clay, allowing it to dry, and then heating it
to remove the molten wax through a tiny hole. The resulting hollow mold was
filled with molten metal taking the shape of the original object.
• Bronze statues of both humans and animals were created with notable examples
including the famous 'Dancing Girl' statue and the artistic buffalo and goat
figures.
• Bronze casting was prevalent across major centers of the Indus Valley Civilization,
with continuous traditions seen in later Harappan and Chalcolithic sites. These
sites produced remarkable metal-cast sculptures of human and animal figures,
showcasing the enduring tradition of figure sculpture throughout the ages.
Dancing Girl
One of the best known artefacts from the Indus
Valley is this approximately four-inch-high
copper figure of a dancing girl. Found in
Mohenjodaro, this exquisite casting depicts a
girl whose long hair is tied in a bun. Bangles
cover her left arm, a bracelet and an amulet or
bangle adorn her right arm, and a cowry shell
necklace is seen around her neck. Her right
hand is on her hip and her left hand is clasped
in a traditional Indian dance gesture. She has
large eyes and flat nose. This figure is full of
expression and bodily vigour and conveys a lot
of information
The Bull
This bronze figure of a bull from
Mohenjodaro deserves
mention. The massiveness of
the bull and the fury of the
charge are eloquently
expressed. The animal is shown
standing with his head turned
to the right and with a cord
around the neck.
Terracotta

• The Indus Valley people made terracotta figures but compared to the stone and
bronze statues the terracotta representations of human form are crude. They are
more realistic in Gujarat sites and Kalibangan.
• The most important among the Indus figures are the ones representing the
mother goddess.
• In terracotta, we also find a few figurines of bearded males with coiled hair, their
posture rigidly upright, legs slightly apart, and the arms parallel to the sides of the
body. The repetition of this figure in exactly the same position would suggest that
he was a deity.
• A terracotta mask of a horned deity has also been found.
• Toy carts with wheels, whistles, rattles, birds and animals, gamesmen and discs
were also rendered in terracotta
Seals

• The Indus Valley Civilization produced numerous seals made of steatite, agate,
chert, copper, faience, and terracotta.
• The seals featured realistic depictions of animals, including unicorn bulls,
rhinoceroses, tigers, elephants, bison, goats, buffaloes, and more.
• Seals served primarily as commercial tools but were also used as amulets or
identification cards.
• The standard Harappan seal was a 2x2 square-inch plaque made of soft river
stone, typically engraved with a pictographic script yet to be deciphered.
• Seals were found in gold and ivory, displaying a wide variety of motifs, often
featuring animals like bulls, elephants, tigers, goats, and monsters. Some seals
depicted trees or human figures.
Seals

• The Pashupati Seal, a remarkable example, featured a cross-legged human figure


surrounded by elephants, tigers, rhinoceroses, buffaloes, and antelopes.
• Copper tablets with engraved animal or human figures and inscriptions on one or
both sides were also discovered, potentially functioning as amulets.
• The inscriptions on seals varied, while those on copper tablets seemed to be
associated with the depicted animals.
• These artifacts date from 2500 to 1500 BCE and were found in significant
numbers at sites like Mohenjodaro, representing the artistic and cultural
achievements of the Indus Valley Civilization.
Unicorn Seals
Pottery
• The pottery from the Indus Valley Civilization provides insights into the evolution
of design motifs, shapes, and styles.
• Wheel-made pottery is predominant, while hand-made pottery is less common.
• Plain pottery, made of red clay with or without a fine red or grey slip, is more
prevalent than painted ware.
• Knobbed ware with rows of knobs and black painted ware with geometric and
animal designs are notable examples.
• Polychrome pottery is rare and features geometric patterns in red, black, green,
white, and yellow.
Pottery

• Incised ware is limited to the bases of pans and dishes of offering stands.
• Perforated pottery with large holes at the bottom and small holes on the walls
was likely used for straining liquor.
• Pottery for household use exhibits a wide variety of shapes and sizes, favoring
graceful curves over straight or angular forms.
• Miniature vessels, often less than half an inch tall, showcase exceptional
craftsmanship.
Bead and Ornaments
• Harappan men and women adorned themselves with a wide variety of
ornaments made from various materials.
• Commonly worn ornaments included necklaces, fillets, armlets, and finger-rings
for both sexes.
• Women additionally wore girdles, earrings, and anklets.
• Jewellry hoards found at Mohenjodaro and Lothal contained gold and semi-
precious stone necklaces, copper bracelets and beads, gold earrings and head
ornaments, faience pendants and buttons, as well as steatite and gemstone
beads.
• Ornaments were skillfully crafted, and some burials at Farmana in Haryana
revealed that bodies were buried with ornaments.
• The bead industry was well-developed, with evidence of factories at Chanhudaro
and Lothal.
Beads and Ornaments

• Beads were made from various materials such as cornelian, amethyst, jasper,
crystal, quartz, steatite, turquoise, lapis lazuli, metals like copper, bronze, and
gold, as well as shell, faience, and terracotta.
• Beads came in different shapes including disc-shaped, cylindrical, spherical,
barrel-shaped, and segmented.
• Some beads were made by combining multiple stones or by attaching gold
covers.
• Beads were decorated through incising, painting, or etching designs onto them,
showcasing great technical skill in their manufacturing.
• The Harappan people crafted naturalistic models of animals, particularly monkeys
and squirrels, which were used as pin-heads and beads.
Fashion
• Spinning of cotton and wool was common, indicated by the discovery of
numerous spindles and spindle whorls in Indus Valley houses.
• Both the rich and the poor practiced spinning, as evidenced by finding whorls
made of expensive faience as well as cheap pottery and shell.
• Men and women wore two separate pieces of attire similar to the dhoti and
shawl, with the shawl covering the left shoulder and passing below the right
shoulder.
• The people of the Indus Valley displayed a fashion-conscious attitude, with
different hairstyles and the popularity of wearing beards.
• Cosmetics such as cinnabar, face paint, lipstick, and collyrium (eyeliner) were
known and used.
Arts Of The Mauryan Period
• In the sixth century BCE, Buddhism and Jainism emerged as significant religious
and social movements in the Gangetic valley, challenging the Hindu varna and jati
systems.
• Magadha became a powerful kingdom, eventually exerting control over other
regions. By the fourth century BCE, the Mauryas rose to power, ruling a large part
of India by the third century BCE.
• Ashoka, the influential Mauryan king, supported the shraman tradition.
• Religious practices were diverse, encompassing worship of Yakshas and mother-
goddesses.
• Buddhism became the most popular movement, absorbing elements of Yaksha
worship.
• Overall, this period marked as one of the transformative era of religious and
social change in ancient India.
Pillars, Sculptures and Rock cut Architecture
• Construction of stupas and viharas became part of the Buddhist tradition.
• Apart from stupas and viharas, stone pillars, rock-cut caves, and monumental
figure sculptures were carved at several places.
• Mauryan pillars are rock-cut, displaying the carver's skills, while Achamenian
pillars are constructed in pieces.
• Stone pillars with inscriptions were erected throughout the Mauryan Empire,
featuring capital figures like bulls, lions, and elephants.
• The Lion Capital at Sarnath, a Mauryan pillar capital, is the finest example of
Mauryan sculptural tradition and is India's national emblem.
• Monumental images of Yaksha, Yakhinis, and animals, as well as polished surface
terracotta figurines, were found in different parts of India.
• The Lomus Rishi cave at Barabar hills in Bihar features a semicircular chaitya arch
and a carved elephant frieze.
• Stupas, viharas, and chaityas were constructed on a large scale due to the
popularity of Buddhism and Jainism.
• Stupas were constructed over the relics of the Buddha at various places, both
within and outside the Gangetic valley.
• The Bairat stupa in Rajasthan is a grand stupa with a circular mound and a
circumambulatory path.
• Inscriptions mention collective patronage by lay devotees, gahapatis, kings,
guilds, and traders.
• Artisans such as Kanha and Balaka are mentioned in inscriptions, indicating their
involvement in the construction.
• Stupas were elaborately built with additions like railings and sculptural
decorations.
• Buddhist monuments depicted events from the life of Buddha and Jataka stories
using synoptic, continuous, and episodic narrative techniques.
Lion Capital, Sarnath

• The Lion Capital discovered at Sarnath is one of the finest


examples of Mauryan sculpture.
• Built by Ashoka in commemoration of the first sermon by
the Buddha at Sarnath.
• The capital originally consisted of five parts: shaft, lotus
bell base, drum with four animals, addorsed lions, and
crowning Dharamchakra wheel.
• The crowning wheel is broken and displayed in the site
museum at Sarnath.
• The capital without the wheel and lotus base is the
National Emblem of Independent India.
Lion Capital, Sarnath

• The capital has four lions seated back to back on a circular abacus.
• The lions are impressive and massive, with strong facial musculature and naturalistic
depiction of lips.
• The lions appear as if they have held their breath, with sharp lines defining the mane.
• The sculpture surface is heavily polished, typical of the Mauryan period.
• The abacus depicts a chakra with twenty-four spokes and animals (bull, horse,
elephant, lion) finely carved between each chakra.
• Each animal figure displays movement in the limited space of the circular abacus.
• The circular abacus is supported by an inverted lotus capital with sculpted petals and
neatly carved curved planes.
• The Lion Capital at Sanchi is in a dilapidated condition.
• The motif of lion-capital-pillar continued in the subsequent period.
Didargunj Yakshini

• The life-size standing image of a Yakshini with a


chauri (flywhisk) is located in Didargunj near
modern Patna.
• The sculpture is made of sandstone with a
polished surface and is kept in the Patna
Museum.
• It is a tall, well-proportioned, free-standing
sculpture in round.
• The Yakshini holds the chauri in the right hand,
while the left hand is broken.
• The sculpture demonstrates sophistication in
form and medium.
• The sculptor's sensitivity towards the round
muscular body is evident.
• The face has round, fleshy cheeks, with a relatively small neck, sharp eyes, nose, and
lips.
• Properly rendered folds of muscles are visible.
• The necklace beads are in full round, hanging to the belly.
• The garment tightly wrapped around the belly creates the effect of a bulging belly.
• The lower garment is carefully rendered, with protruding lines representing folds on
the legs.
• The middle band of the garment falls till the feet, while thick bell-ornaments adorn the
feet.
• The image stands firmly on its legs, depicting heaviness in the torso through heavy
breasts.
• The back of the sculpture is equally impressive, with tied hair in a knot and a bare back.
• Drapery at the back covers both legs.
• The chauri (flywhisk) in the right hand is depicted with incised lines continued on the
back of the image.
Post Mauryan trends in Indian Art and Architecture
Bharhut
• Bharhut sculptures in the Mauryan period are tall and resemble Yaksha and
Yakhshini images.
• The sculptural volume is in low relief with a focus on maintaining linearity.
• Images stick to the picture plane, but in narrative panels, a tilted perspective
creates an illusion of three-dimensionality.
• Clarity in the narrative is enhanced by selecting main events.
• At Bharhut, narrative panels show fewer characters initially, but as time
progresses, others also start appearing in the picture space.
• More than one event at one geographical place may be depicted in the picture
space, or only a single main event may be shown.
• Sculptors utilize the available space to the maximum extent.
• Folded hands in the narratives and single figures of the Yakhshas and Yakshinis
are shown flat, clinging to the chest.
• In later times, hands are shown with a natural projection against the chest.
• Artisans working at a collective level had to understand the method of carving,
initially focusing on dressing the surface of stone slabs.
• Later, the human body and other forms were sculpted.
• Shallow carving of the picture surface made it difficult to project hands and feet,
resulting in folded hands and awkward positions of the feet.
• The body and arms exhibit a general stiffness, but gradually, sculptures with deep
carvings, pronounced volume, and a very naturalistic representation of human
and animal bodies emerged.
• Bharhut, Bodhgaya, Sanchi Stupa-2, and Jagayyapetta are good examples of such
sculptures.
• Narrative reliefs at Bharhut effectively use the pictorial language to communicate
stories.
• One such narrative depicts Queen Mayadevi's (mother of Siddhartha Gautam)
dream, showing a descending elephant and the queen reclining on a bed.
• Another example is the depiction of the Jataka story of Ruru, where the
Boddhisattva deer rescues a man on his back.
• In the same picture frame, the king with his army is shown about to shoot an
arrow at the deer, and the rescued man is also depicted, pointing a finger at the
deer.
• Jataka stories became part of stupa decoration.
• With the rise in the construction of stupas in various parts of the country,
regional stylistic variations also began to emerge.
• One characteristic of male images from the first and second centuries BCE is the
knotted headgear, which is consistent in many sculptures.
• Some Bharhut sculptures are displayed at the Indian Museum, Kolkata.
Sanchi

• Sanchi Stupa-1, Mathura, and Vengi in Andhra Pradesh (Guntur District) represent
the next phase of sculptural development.
• Stupa-1 at Sanchi has both upper and lower circumambulatory paths
(pradakshinapatha).
• The stupa features four beautifully decorated toranas (gateways) depicting events
from the life of the Buddha and the Jatakas.
• Figure compositions are in high relief, filling up the entire space.
• Depictions of postures become naturalistic, and there is no stiffness in the body.
• Heads have considerable projection in the picture space, reducing rigidity in the
contours.
• Images are given movement and narration becomes more elaborate.
• Carving techniques at Sanchi appear more advanced than those at Bharhut.
• Symbols continue to be used to represent the Buddha and the Manushi Buddhas
(past Buddhas).
• Sanchi Stupa-1 showcases more elaborate narratives, although the depiction of
the dream episode remains simple with the reclining image of the queen and the
elephant at the top.
• Historical narratives, such as the siege of Kushinara, Buddha's visit to Kapilavastu,
and Ashoka's visit to the Ramgrama Stupa, are carved with considerable details.
• In Mathura, images from this period possess the same quality but differ in the
depiction of physiognomic details.
Stupa-1, Sanchi
Stupa-1, Sanchi
Mathura, Sarnath and Gandhara Schools

• From the 1st century CE onwards, Gandhara (now in Pakistan), Mathura in


northern India, and Vengi in Andhra Pradesh became important centers of art
production.
• Mathura and Gandhara were significant for the emergence of the human form of
the Buddha in sculpture.
• The sculptural tradition in Gandhara was influenced by Bactria, Parthia, and the
local Gandhara tradition.
• The strong local sculptural tradition in Mathura spread to other parts of northern
India, exemplified by the stupa sculptures found at Sanghol in Punjab.
• Mathura predominantly features images of Vishnu and Shiva in addition to
Buddhist images, represented by their respective weapons (ayudhas).
• Carving of large images in Mathura displays boldness, projecting the volume of
the images out of the picture plane.
• The faces of the sculptures are round and smiling, with reduced heaviness in the
sculptural volume and clearly visible garments covering the left shoulder.
• Mathura sculptures include images of the Buddha, Yakshas, Yakshinis, Shaivite
and Vaishnavite deities, and portrait statues.
• In the 2nd century CE, Mathura images become more sensual and fleshier.
• In the 3rd century CE, treatment of sculptural volume changes, reducing extreme
fleshiness and introducing movement in posture through increased distance
between legs and body bents.
• Surface softness continues to be refined.
• The trend of reduced massiveness and fleshiness continues in the 4th century CE,
with more tightened flesh and reduced volume of drapery.
• In the 5th and 6th centuries CE, drapery is integrated into the sculptural mass,
displaying a transparent quality in the robes of Buddha images.
• Two important schools of sculptures in northern India are Mathura, Sarnath, and
Kosambi.
• Buddha images in Sarnath have plain transparent drapery covering both
shoulders, with minimal ornamentation in the halo.
• Mathura Buddha images continue to depict folds of drapery and feature highly
decorated halos.
• Museums in Mathura, Sarnath, Varanasi, New Delhi, Chennai, Amaravati, etc.
provide opportunities to study early sculptures.
Seated Buddha, Katra Mound, Mathura
•Mathura was a significant center for sculpture production
during the early historic period.
•Many sculptures dating back to the Kushana Period have
been found in Mathura.
•Sculptures from Mathura have a distinct style that sets
them apart from those found in other parts of the country.
•The image of the Buddha from the Katra mound in
Mathura belongs to the second century CE.
•The Buddha is depicted seated in padmasana (cross-
folded legs) with two Boddhisattva attendants.
•The Buddha's right hand is raised in the abhayamudra
(gesture of fearlessness), while the left hand rests on the
left thigh.
•The Buddha's ushanisha (hair knot) is shown with a
vertically raised projection.
Seated Buddha, Katra Mound, Mathura
•Mathura sculptures from this period have a light volume and fleshy bodies.
•The shoulders are broad, and the sanghati (garment) covers only one shoulder, prominently
displaying the left hand.
•The independent volume of the garment is reduced to the body torso.
•The Buddha is seated on a lion throne, flanked by the Padmapani and Vajrapani Boddhisattvas.
•The attendants hold a lotus and a vajra (thunderbolt) and wear crowns.
•The halo around the Buddha's head is large and decorated with simple geometric motifs.
•Above the halo, there are two flying figures with dynamic movement.
•The sculptures show flexibility, replacing earlier rigidity and giving them an earthy look.
•Delicate curves are carved to depict the body.
•The upright posture of the Buddha creates a sense of movement in space.
•The Buddha's face is round with fleshy cheeks, and the belly is sculpted with controlled
musculature.
•Mathura has numerous examples of sculptures from the Kushana Period, but this particular image
is representative and important for understanding the development of the Buddha image in
subsequent periods.
Buddha Head, Taxila
• The Buddha head from Taxila in the Gandhara region, now
in Pakistan, dates back to the second century CE and
belongs to the Kushana Period.
• The image exhibits hybridized pictorial conventions
developed during the Gandhara period.
• It incorporates Greco-Roman elements in its sculpture
treatment.
• The Buddha head features thick, curly hair with sharp and
linear strokes.
• The forehead plane is large with protruding eyeballs, and
the eyes are half-closed.
• The face and cheeks have a non-round shape unlike
images from other parts of India.
• The figures from the Gandhara region have a certain
heaviness.
• The ears, especially the earlobes, are elongated.
Buddha Head, Taxila
• The treatment of form emphasizes linearity with sharp outlines and a smooth surface.
• The image is highly expressive, focusing on the interplay of light and dark.
• The expression of calmness is a central point of attraction.
• The facial modeling enhances the naturalism of three-dimensionality.
• The Gandhara style assimilates traits from Acamenian, Parthian, and Bactrian traditions
into the local tradition.
• The Gandhara images display physiognomic features of the Greco-Roman tradition but
with a distinct treatment of details.
• The development of Buddha images and others in the region is influenced by its unique
geo-political conditions.
• The north-western part of India, now Pakistan, has a long history of continuous
habitation.
• The Gandhara region has yielded a large number of images, including narratives of the
life of the Buddha, Jataka stories, and Buddha and Bodhisattva depictions.
Buddhist Mounuments of South India
• Vengi in Andhra Pradesh houses several stupa sites, including Jagayyapetta,
Amaravati, Bhattiprolu, Nagarjunkonda, and Goli.
• Amaravati Stupa in Amaravati has a mahachaitya with narrative sculptures, and
its pradakshinapatha is enclosed within a vedika.
• The domical stupa structure of Amaravati is covered with relief stupa sculptural
slabs, although the torana has disappeared over time.
• Construction activity in the Amaravati Stupa is evident from the third century
BCE, but it was most developed in the first and second centuries CE.
• The early phase of Amaravati Stupa lacks Buddha images, but they appear in the
later phase during the second and third centuries CE.
• Sculptural forms in the area exhibit intense emotions, slender figures with
movement, and tribhanga (three-bent) body postures.
• The sculptural compositions at Amaravati are more complex than at Sanchi, with
flexible linearity, dynamic movement, and three-dimensional space.
• Various narratives from the life of the Buddha and Jataka stories are profusely
depicted, including birth scenes and dream sequences.
• Sculptures in Nagarjunkonda and Goli from the third century CE show reduced
animated movement but maintain integral protruding surfaces.
• Independent Buddha images are found at Amaravati, Nagarjunkonda, Guntapalle,
and other sites like Anakapalle and Sannati in Karnataka.
• While many stupas were constructed, structured temples, viharas, and chaityas
also existed, although no structured chaitya or vihara has survived.
• Along with Buddha images, sculptures of Bodhisattvas like Avalokiteshvara,
Padmapani, Vajrapani, Amitabha, and Maitreya Buddha were created.
• With the rise of Vajrayana Buddhism, Bodhisattva images were added to
represent virtues or qualities as part of the Buddhist religious principles.
Early Temples
• Construction of Brahmanical temples and the creation of images of gods began alongside
the continued construction of stupas.
• Temples were often adorned with images of gods, and myths from the Puranas became
part of the narrative representation of the Brahmanical religion.
• Each temple had a principal image of a god.
• Temples were categorized into three types: sandhara (without circumambulatory path),
nirandhara (with circumambulatory path), and sarvatobhadra (accessible from all sides).
• Some important temple sites from this period include Deogarh in Uttar Pradesh, Eran,
Nachna-Kuthara, and Udaygiri near Vidisha in Madhya Pradesh.
• These temples were simple structures consisting of a veranda, a hall, and a shrine at the
rear.
• Outside the Gangetic valley, Devnimori in Gujarat is an important stupa site.
• In the subsequent centuries, sculptures showed little variation, and slender images with
transparent drapery remained a dominant aesthetic sensibility.
Cave Tradition in Western India
• Three main architectural types of caves were executed: apsidal vault-roof chaitya halls,
apsidal vault-roof pillarless halls, and flat-roofed quadrangular halls with a circular
chamber at the back.
• The front of the chaitya halls is dominated by a semi-circular chaitya arch, except for
some cases like Kondivite.
• All chaitya caves have a stupa at the back.
• In next century, modifications were made to the standard apsidal vault-roof plan,
resulting in rectangular halls with a stone-screen wall facade.
• The standard chaitya halls continued to be built in subsequent periods, such as at Karla.
• Karla chaitya hall is the largest rock-cut chaitya hall and is decorated with human and
animal figures.
• Kanheri Cave No. 3 shows further elaboration of the Karla chaitya hall plan.
• The quadrangular flat-roofed variety became the most preferred design for caves.
Cave Tradition in Western India
• Viharas are excavated in all cave sites, consisting of a veranda, a hall, and cells around the
walls.
• Early vihara caves feature interior decorative motifs like chaitya arches and vedica
designs.
• Nashik Cave Nos. 3, 10, and 17 showcase distinct facade designs.
• Major cave sites include Ajanta, Pitalkhora, Ellora, Nashik, Bhaja, Junnar, Karla, and
Kanheri.
• Ajanta, Ellora, and Kanheri caves are still active and flourishing.
• The absence of Buddha images initially led to the assumption that the caves belonged to
the Thervadins, but the discovery of the Konkan Maurya inscription proved ongoing cave
activity and the presence of Buddha images.
• Many caves have been converted into modern Hindu shrines and become popular
worship sites.
Ajanta

• Ajanta is the most famous cave site located in Aurangabad District, Maharashtra.
• It has twenty-nine caves, including four chaitya caves from the second and first
century BCE (Cave Nos. 10 and 9) and the fifth century CE (Cave Nos. 19 and 26).
• Ajanta features large chaitya-viharas adorned with sculptures and paintings.
• It is the only surviving example of first-century BCE and fifth-century CE paintings.
• The caves at Ajanta and in western Deccan lack precise chronology due to the
absence of known dated inscriptions.
• Cave Nos. 10, 9, 12, and 13 belong to the early phase, while Caves Nos. 11, 15, 6
(upper and lower), and 7 belong to the phase earlier than the late fifth century
CE.
Ajanta
• The remaining caves date from the late fifth century CE to the early sixth century CE,
with elaborate carvings found in the apsidal-vault-roof chaitya Cave Nos. 19 and 26.
• Ajanta's vihara-chaitya caves consist of a pillared veranda, hall, and cells along the walls,
with a main Buddha shrine on the back wall.
• Some vihara caves, such as Cave Nos. 5, 14, 23, 24, 28, and 29, are unfinished.
• Notable patrons at Ajanta include Varahadeva, Upendragupta, Buddhabhadra, and
Mathuradasa.
• Paintings in Ajanta exhibit typological variations, with the use of outward projections and
sharp lines.
• Ajanta's early phase caves, particularly Cave Nos. 9 and 10, depict figures with heavy
proportions arranged linearly and painted with naturalism.
• The second phase of paintings can be observed in Cave Nos. 10 and 9, showcasing
different Buddha figures than those painted in the fifth century CE.
Ajanta
• The development of paintings at Ajanta correlates with the date of cave excavations.
• Cave Nos. 16, 17, 1, and 2 represent the next stage of development in painting, with
orderly and naturalistic compositions integrated with sculptures.
• Themes of the paintings include events from the life of the Buddha, Jatakas, and
Avadanas, with some covering entire cave walls.
• Paintings in Cave No. 10 depict the Chaddanta Jataka with events grouped by
geographical locations.
• Cave No. 1 features the famous Padmapani and Vajrapani paintings.
• Paintings in Cave No. 2 show influences from Vengi sculptures and the Vidarbha
sculptural tradition.
• The subsequent development of the painting tradition is discussed in the next chapter.
• The paintings in Cave No. 1 at Ajanta are of good quality and are better
preserved.
• Different typological and stylistic variations can be observed in the paintings of
Ajanta, indicating the involvement of different guilds of artisans over the
centuries.
Padmapani Boddhisattva
Ajanta Cave 01
•The painting is located on the back wall of the interior
hall before the shrine-antechamber in Cave No. 1 at
Ajanta. It dates back to the late fifth century CE.
•The depicted figure is a Boddhisattva holding a lotus
(padma).
•The Boddhisattva has large shoulders and three
bents in the body, creating movement in the picture
space.
•The modeling of the figure is soft, and the outlines
merge with the body volume, giving a three-
dimensional effect.
•The Boddhisattva wears a detailed crown, and the
head is slightly bent to the left.
•The eyes are half-closed and slightly elongated, while
the nose is sharp and straight.
Padmapani Boddhisattva Ajanta Cave 01
•Light colors are used to create a three-dimensional effect on the projected planes of
the face.
•The beaded necklace exhibits similar features to the facial rendering.
•Broad and expanded shoulders contribute to the heaviness of the body, which has a
relatively round torso.
•Delicate and rhythmic lines define the contours of the body.
•The right hand holds a lotus, while the left hand is extended into space.
•The foreshortened right hand adds solidity and density to the image.
•The thread over the torso is depicted with fine spiral lines, indicating its dimensions.
•Equal attention is given to each part of the body.
•The Boddhisattva is surrounded by small figures.
•Light red, brown, green, and blue colors are used in the painting.
•Various details, such as nose projections, incised end of lips, lower lip projection, and
a small chin, contribute to the overall effect of solidity in the figure composition.
Paintinf of Mahajanak Jataka
Ajanta Cave 01
•Cave No. 1 has many interesting paintings of
Buddhist themes such as Mahajanak Jataka,
Umag Jataka, etc.
• The Mahajanak Jataka is painted on the
entire wall side and is the biggest narrative
painting.
•It may be observed that the paintings of
Padmapani and Vajrapani and the
Bodhisattvas are painted as shrine guardians.
•Similar such iconographic arrangement is also
observed in other caves of Ajanta. However
Padmapani and Vajrapani in Cave No. 1 are
among the best survived paintings of Ajanta.
Mara Vijaya
Ajanta Cave 26
•The theme of Mara Vijaya has been painted in the
caves of Ajanta. This is the only sculptural
representation sculpted on the right wall of Cave No.
26.
•It is sculpted near the colossal Buddha image of
Mahaparinibbana.
•The panel shows the image of the Buddha in the
centre surrounded by Mara’s army along with his
daughter.
•The event is part of the enlightenment. It is a
personification of the commotion of mind which the
Buddha went through at the time of enlightenment.
•Mara represents desire.
Mara Vijaya
Ajanta Cave 26 •The relief sculptural panel depicts a dialogue between
the Buddha and Mara.
•The Buddha is shown with his right hand indicating
towards the earth as a witness to his generosity.
•The panel exhibits a highly animated and matured
sculptural style at Ajanta.
•The composition is complex, with highly voluminous
images arranged dynamically in the picture space.
•Mara is shown on the right side, accompanied by his
army, which includes various people and some with
grotesque animal faces.
•The lower base of the panel features dancing figures
with forward bulging waists and one figure with
expanded hands in a dancing posture.
•Mara is depicted contemplating on the left lower end,
considering how to disturb Siddhartha (the Buddha
before enlightenment).
Mara Vijaya
Ajanta Cave 26

•The army of Mara marches towards the Buddha in the upper half of the panel, while
the lower half shows the departing army giving adorations.
•The central figure is the Buddha seated in padmasana, and a tree with dense leaves
is depicted in the background.
•Some facial features of the Mara army resemble sculptures from Vidarbha.
•The artisans at Ajanta worked in guilds, and their stylistic affiliations can be identified
through such features.
•This relief panel is the largest at Ajanta and showcases a unique and complex
arrangement of figures.
•While there are several big images in the caves of Ajanta, this panel stands out for its
intricate composition.
•Similar arrangements of dancing figures can be observed in painted panels at the
Aurangabad caves.
Ellora

• Ellora is an important cave site located in Aurangabad District, approximately 100


kilometers from Ajanta.
• It consists of thirty-two caves representing Buddhist, Brahmanical, and Jain traditions.
• Ellora is unique in terms of its art-historical significance, featuring monasteries associated
with Buddhism, Brahmanism, and Jainism from the fifth to eleventh century CE.
• The caves of Ellora showcase a stylistic eclecticism, combining various artistic styles in
one place.
• The site demonstrates the ongoing differences between Buddhism and Brahmanism.
• There are twelve Buddhist caves at Ellora, featuring images related to Vajrayana
Buddhism, such as Tara, Mahamayuri, Akshobhya, Avalokiteshwara, Maitreya, Amitabha,
and more.
• The Buddhist caves are large in size and range from single to triple storeys, with massive
pillars.
Ellora

• The shrine Buddha images are sizable and often accompanied by figures of Padmapani
and Vajrapani.
• Cave No. 12, a triple-storey excavation, contains images of Tara, Avalokiteshwara,
Manushi Buddhas, Vairochana, Akshobhya, Ratnasambhava, Amitabha, Amoghsiddhi,
Vajrasatva, and Vajraraja.
• Among the Brahmanical caves, Cave No. 14 is the only double-storey cave. Pillar designs
evolve from the Buddhist caves and become more ornate in the Jain caves from the ninth
century CE onward.
• Brahmanical caves (Caves Nos. 13-28) feature numerous sculptures representing both
Shiva and Vishnu, including their various forms according to Puranic narratives.
• Shaivite themes depicted include Ravana shaking Mount Kailash, Andhakasuravadha, and
Kalyanasundara.
• Vaishnavite themes portray the different avatars of Vishnu.
Ellora

• The sculptures at Ellora are monumental, displaying protruding volumes and creating a
deep recession in the picture space.
• The sculptures exhibit sophistication in handling sculptural volume and were carved by
various guilds from different regions such as Vidarbha, Karnataka, and Tamil Nadu.
• Cave No. 16, known as Kailash leni, is a remarkable rock-cut temple carved out of a single
rock, which will be discussed later.
• Notable Shaivite caves include Cave No. 29 and Cave No. 21, with Cave No. 29 having a
similar plan to the main cave at Elephanta.
• Cave Nos. 29, 21, 17, 14, and 16 showcase astonishing sculptural quality characterized by
monumentality and vigorous movements in the picture space.
Elephanta caves and other sites

• The Elephanta Caves near Mumbai were originally a Buddhist site but later became
dominated by the Shaivite faith.
• The sculptures in Elephanta Caves exhibit slenderness in the body and utilize stark light
and dark effects.
• Another significant cave site is Bagh, located near Indore in Madhya Pradesh.
• Rock-cut caves continued to be created in the Deccan region, including Maharashtra and
Karnataka (Badami and Aiholi), under the patronage of the Chalukyas.
• Andhra Pradesh, particularly in the area of Vijayawada, also features rock-cut caves.
• Tamil Nadu, mainly at Mahabalipuram, showcases rock-cut caves created under the
patronage of the Pallavas.
• The development of art history in India after the sixth century relied more on political
patronage rather than collective public patronage seen in earlier historic periods.
Elephanta caves and other sites
• Buddhist caves have been excavated in eastern India, particularly in the coastal
regions of Andhra Pradesh and Odisha.
• Guntapalle in Eluru district, Andhra Pradesh, is one of the main cave sites in the
region. It features structured stupas, viharas, and caves excavated in the hills.
• The Guntapalle chaitya cave is circular with a stupa in the circular hall and a
chaitya arch at the entrance. It is relatively small compared to caves in western
India.
• Vihara caves have also been excavated in Guntapalle, despite their small
dimensions. The main vihara caves are rectangular with vaulted roofs and
decorated with chaitya arches on the exterior. They date back to the second
century BCE.
• Rampaerrampallam is another important cave site in Andhra Pradesh, featuring
moderate small excavations and rock-cut stupas on the hillock.
Elephanta caves and other sites
• Anakapalli near Vishakhapatnam has caves and a large rock-cut stupa carved out
of the hillock during the fourth-fifth centuries CE. It is known for having the
largest rock-cut stupas in the country.
• Odisha also has a rock-cut cave tradition. The Udaigiri-Khandagiri caves near
Bhubaneswar are early examples with inscriptions of Kharavela kings. These caves
were meant for Jain monks and include numerous single-cell excavations.
• Some of the caves in Odisha have been carved in huge independent boulders,
resembling the shape of animals. There are also larger caves with pillared
verandas and cells at the back, decorated with chaitya arches and narratives.
• The Udaigiri-Khandagiri cave complex features voluminous figures that exhibit
freedom of movement in the picture space, showcasing excellent qualitative
carving.
• Some caves in the complex were excavated in the eighth-ninth centuries CE
Maheshmurti, Elephanta
Maheshmurti, Elephanta
QnA
1.Describe the physical and aesthetic features of Sanchi Stupa-I.

• Sanchi Stupa-I is a large hemispherical dome-shaped structure located in Sanchi,


Madhya Pradesh, India.
• It is made of stone and measures approximately 16.4 meters in diameter and 16.4
meters in height.
• The stupa is built on a raised platform, called a harmika, which is adorned with railing
and four gateways (toranas) in each cardinal direction.
• The gateways of Stupa-I are intricately carved with scenes from the life of the Buddha
and various motifs depicting Buddhist symbols and teachings.
• The stupa has a central core, called the anda, which is surrounded by layers of brickwork
and stone. The entire structure is covered with a layer of plaster.
• The harmika of Stupa-I is crowned with an umbrella-like structure, known as the chatra,
symbolizing the spiritual significance of the stupa.
• The aesthetic features of Sanchi Stupa-I include the harmonious proportions, the
intricate carvings on the gateways, and the overall simplicity and elegance of its design.
• The sculptures and carvings on the gateways of Stupa-I represent a transition from the
early Mauryan style to the more elaborate and narrative-rich style seen in later Buddhist
art.
2. Analyse the stylistic trends of the sculptures in North India during the fifth and sixth centuries.

• During the fifth and sixth centuries, North Indian sculptures witnessed a shift towards more
naturalistic and dynamic representations.
• The sculptures showed a blend of indigenous Indian traditions with influences from foreign
artistic styles, such as the Greco-Roman and Gandharan styles.
• The figures became more three-dimensional, with realistic proportions and anatomical details.
• The facial expressions became more expressive, conveying emotions and inner states of the
figures.
• The hairstyles and costumes of the sculptures reflected the fashion trends of the time and varied
across different regions and dynasties.
• Sculptures of deities, including Hindu gods and goddesses, displayed a sense of divine grace and
majesty.
• The narrative reliefs on temple walls and stupas depicted stories from mythology, religious texts,
and historical events, providing a visual narrative for the viewers.
• The sculptural style during this period also witnessed the emergence of regional variations and
local artistic traditions, with each region contributing its own distinct characteristics to the
sculptures.
3. How did cave architecture develop in different parts of India, from cave shelters to the monolitic
temple at Ellora?
• Cave architecture in India evolved from simple cave shelters used by early humans to more elaborate
and purpose-built cave complexes.
• Initially, caves were utilized as natural shelters and habitation spaces. Over time, they were
transformed into sacred spaces for religious and artistic purposes.
• The earliest known examples of rock-cut caves in India are found in the Barabar Hills of Bihar, dating
back to the Mauryan period (3rd century BCE). These caves were mainly used by ascetics and monks
for meditation and retreat.
• The next stage of cave development witnessed the carving of caves for Buddhist monastic complexes,
such as those at Ajanta and Ellora. These caves included monasteries, prayer halls, and chaitya halls
(cave temples) for communal worship.
• The Ajanta caves, dating from the 2nd century BCE to the 6th century CE, are renowned for their mural
paintings depicting Buddhist narratives and themes.
• The pinnacle of cave architecture is seen at Ellora, where the monolithic Kailasa Temple was carved out
of a single rock in the 8th century CE. This temple exemplifies the architectural and sculptural
achievements of the time.
• Cave architecture in other parts of India, such as Maharashtra, Karnataka, Andhra Pradesh, and Tamil
Nadu, also flourished, with the creation of rock-cut temples and monasteries under the patronage of
different dynasties.
• These cave complexes served as important religious, artistic, and educational centers, preserving and
showcasing the cultural heritage of the region.
4. Why are the mural paintings of Ajanta renowned?

• The mural paintings of Ajanta are renowned for their artistic excellence, technical skill, and their
historical and cultural significance.
• These paintings, created between the 2nd century BCE and the 6th century CE, depict various
themes, including the life of the Buddha, Jataka tales (stories of the Buddha's previous lives), and
courtly and social scenes of ancient India.
• The paintings showcase a high level of artistic sophistication, with a mastery of composition, color
usage, and intricate detailing.
• The artists employed techniques such as fresco secco (painting on dry plaster) and tempera
(painting with pigments mixed with a water-soluble binder) to create the vibrant and enduring
paintings.
• The mural paintings of Ajanta provide valuable insights into ancient Indian society, including its
religious beliefs, social customs, and artistic traditions.
• These paintings are considered a significant milestone in the development of Indian art,
influencing subsequent artistic styles and traditions.
• The preservation and restoration efforts undertaken over the years have helped to safeguard and
showcase the beauty and cultural significance of the Ajanta mural paintings, making them a
UNESCO World Heritage site.
5. Describe the padmapani boddhistava paiting of ajanta caves.

The Padmapani Bodhisattva painting in Ajanta Caves is a significant representation of Buddhist art
and iconography. It is located on the back wall of the interior hall before the shrine-antechamber in
Cave No. 1.
The painting depicts Padmapani Bodhisattva, a compassionate and enlightened being who has
chosen to postpone his own enlightenment to guide and assist others in their spiritual journeys.
Here are the key features of the painting:
• Iconography: Padmapani Bodhisattva is depicted as a male figure, standing gracefully with a slight
bend to the left. He is shown holding a padma (lotus) in his right hand, a symbol of purity and
enlightenment. The left hand is extended in the space, possibly in a gesture of compassion or
giving.
• Physical Attributes: The Padmapani Bodhisattva is depicted with a slender and graceful body. He
has broad, expanded shoulders and a relatively round torso, creating a sense of solidity and
weight in the figure. The modeling of the body is soft, with gentle curves and flowing lines.
• Facial Features: The Bodhisattva's face reflects a sense of serenity and tranquility. The eyes are
half-closed and slightly elongated, while the nose is sharp and straight. The head is adorned with
a large crown, intricately rendered with detailed patterns and ornamentation.
• Three-dimensionality: The artist skillfully employs the use of shading and merging outlines with
the body volume to create a sense of three-dimensionality. Light colors are applied to the
projected planes of the face, enhancing the sculptural quality and depth of the figure.
• Surrounding Figures: The Padmapani Bodhisattva is surrounded by small figures, possibly
devotees or celestial beings, who are shown in adoration or paying homage to the Bodhisattva.
• Color Palette: The painting utilizes a range of colors, including light red, brown, green, and blue.
These colors are applied to different elements of the painting, such as the Bodhisattva's body, the
lotus, and the beaded necklace, adding vibrancy and visual interest to the composition.
Later Mural Traditions
• Even after Ajanta, very few sites with paintings have survived which provide
valuable evidences to reconstruct the tradition of paintings.
• It may also be noted that the sculptures too were plastered and painted.
• The tradition of cave excavations continued further at many places where
sculpting and painting were done simultaneously.
• Even today we observe that mural painting on interior and exterior walls of
houses in villages or havelis is prevalent in different parts of the country. These
paintings are usually made by women either at the time of ceremonies or
festivals or as a routine to clean and decorate the walls.
• Some of the traditional forms of murals are pithoro in parts of Rajasthan and
Gujarat, Mithila painting in northern Bihar’s Mithila region, warli paintings in
Maharashtra, or simply paintings on the walls, be it in a village of Odisha or
Bengal, Madhya Pradesh or Chhattisgarh.
Badami
• Badami is in the State of Karnataka. Badami was the capital of the western
Chalukyan dynasty which ruled the region from 543 to 598 CE.
• With the decline of the Vakataka rule, the Chalukyas established their power in
the Deccan.
• The Chalukya king, Mangalesha, patronised the excavation of the Badami caves.
He was the younger son of the Chalukya king, Pulakesi I.
• The inscription in Cave No.4 mentions the date 578–579 CE, describes the beauty
of the cave and includes the dedication of the image of Vishnu. Thus it may be
presumed that the cave was excavated in the same era and the patron records his
Vaishnava affiliation. Therefore, the cave is popularly known as the Vishnu Cave.
• Only a fragment of the painting has survived on the vaulted roof of the front
mandapa.
• Paintings in this cave depict palace scenes. One shows Kirtivarman, the son of
Pulakesi I and the elder brother of Mangalesha, seated inside the palace with his
wife and feudatories watching a dance scene.
Badami
• Towards the corner of the panel are figures of Indra and his retinue. Stylistically
speaking, the painting represents an extension of the tradition of mural painting
from Ajanta to Badami in South India.
• The sinuously drawn lines, fluid forms and compact composition exemplify the
proficiency and maturity the artists had achieved in the sixth century CE.
• The gracefully drawn faces of the king and the queen remind us of the style of
modelling in Ajanta.
• Their eye sockets are large, eyes are half-closed, and lips are protruding. It is
noteworthy to observe that the contours of different parts of the face create
protruding structures of the face itself. Thus, with simple line treatment artists
could create volume.
Murals under the Pallava, Pandava and Chola Kings
• The tradition of painting extended further down south in Tamil Nadu in the preceding
centuries with regional variations during the regimes of Pallava, Pandya and Chola
dynasties.
• The Pallava kings who succeeded the Chalukya kings in parts of South India, were also
patrons of arts. Mahendravarma I who ruled in the seventh century was responsible for
building temples at Panamalai, Mandagapattu and Kanchipuram.
• The inscription at Mandagapattu mentions Mahendravarman I with numerous titles such
as Vichitrachitta (curious-minded), Chitrakarapuli (tiger among artists), Chaityakari
(temple builder), which show his interest in art activities. The paintings in these temples
too were done at his initiative, though only fragments remain.
• The Panamalai figure of a female divinity is drawn gracefully.
• Paintings at the Kanchipuram temple were patronised by the Pallava king, Rajsimha. Only
traces of paintings remain now which depict Somaskanda. Faces are round and large.
Lines are rhythmic with increased ornamentation when compared with the paintings of
an earlier periods. Depiction of torso still remains like the earlier sculptural tradition but
is elongated.
Murals under the Pallava, Pandava and Chola Kings
• When the Pandyas rose to power, they too patronised art. Tirumalaipuram caves and
Jaina caves at Sittanvasal are some of the surviving examples. A few fragmented layers of
paintings can be seen in Tirumalaipuram. In Sittanavasal, the paintings are visible on the
ceilings of shrines, in verandas, and on the brackets.
• On the pillars of the veranda are seen dancing figures of celestial nymphs. The contours
of figures are firmly drawn and painted in vermilion red on a lighter background. The
body is rendered in yellow with subtle modelling. Supple limbs, expression on the faces
of dancers, rhythm in their swaying movement, all speak of the artists’ skill in creative
imagination in visualising the forms in the architectural context. Their eyes are slightly
elongated and at times protrude off the face. This feature is observed in many
subsequent paintings in the Deccan and South India.
• The tradition of building temples and embellishing them with carvings and paintings
continued during the reign of the Chola kings who ruled over the region from the ninth
to the thirteenth century. But it was in the eleventh century, when the Cholas reached
their zenith of power, that masterpieces of Chola art and architecture began to appear.
Murals under the Pallava, Pandava and Chola Kings
• The temples of Brihadeswara at Thanjavur, Gangaikonda Cholapuram and Darasuram
were built during the reigns of Rajaraja Chola and his son, Rajendra Chola. Though Chola
paintings are seen in Nartamalai, the most important are those in Brihadeswara temple.
• The paintings were executed on the walls of the narrow passage surrounding the shrine.
Two layers of paint were found when they were discovered. The upper layer was painted
during the Nayak period, in the sixteenth century. Thanks to the cleaning of the surface
painting, examples of the great tradition of painting during the Chola Period were
unveiled.
• The paintings show narrations and aspects related to Lord Shiva, Shiva in Kailash, Shiva as
Tripurantaka, Shiva as Nataraja, a portrait of the patron Rajaraja and his mentor Kuruvar,
dancing figures, etc.
Vijayanagara and Nayaka Dynasty Murals
• The paintings of Brihadeswara temple exemplify the stylistic maturity the artists evolved
over the years. Sinuous pre-determined flow of lines, supple modelling of figures,
elongation of the physiognomic features of human figures— all these represent the
perfection the Chola artist had achieved during the period on the one hand and the
phase of transition on the other.
• With the decline of power of the Chola dynasty in the thirteenth century, the
Vijayanagara Dynasty captured and brought under its control the region from Hampi to
Trichy with Hampi serving as its capital.
• Many paintings survive in a number of temples. The paintings at Tiruparakunram, near
Trichy, done in the fourteenth century represent the early phase of the Vijayanagara
style.
• In Hampi, the Virupaksha temple has paintings on the ceiling of its mandapa narrating
events from dynastic history and episodes from the Ramayana and the Mahabharata.
Vijayanagara and Nayaka Dynasty Murals
• Among the important panels are the ones which show Vidyaranya, the spiritual teacher
of Bukkaraya Harsha, being carried in a palanquin in a procession and the incarnations of
Vishnu. The faces of the figures are shown in profile, with large frontal eyes. The figures
have narrow waists.
• In Lepakshi, near Hindupur, in present Andhra Pradesh, there are glorious examples of
Vijayanagara paintings on the walls of the Shiva temple.
• In keeping with the tradition, the Vijayanagara painters evolved a pictorial language
wherein the faces are shown in profile and figures and objects two-dimensionally.
• Lines become still but fluid, compositions appear in rectilinear compartments. These
stylistic conventions of the preceding centuries were adopted by artists in various
centres in South India as can be seen in the paintings of the Nayaka Period.
• Nayaka paintings of the seventeenth and eigtheenth centuries are seen in
Thiruparakunram, Sreerangam and Tiruvarur.
Vijayanagara and Nayaka Dynasty Murals
• In Thiruparakunram, paintings are found of two different periods—of the fourteenth and the
seventeenth century. Early paintings depict scenes from the life of Vardhaman Mahavira.
• The Nayaka paintings depict episodes from the Mahabharata and the Ramayana and also scenes
from Krishna-leela.
• In Tiruvarur, there is a panel narrating the story of Muchukunda.
• In Chidambaram there are panels of paintings narrating stories related to Shiva and Vishnu—
Shiva as bhikshatana murti, Vishnu as Mohini, etc.
• In the Sri Krishna temple at Chengam in Arcot District there are sixty panels narrating the story of
the Ramayana which represent the late phase of Nayaka paintings.
• The examples cited above suggest that Nayaka paintings were more or less an extension of the
Vijayanagara style with minor regional modifications and incorporations. The figures, mostly in
profile, are set against a flat background. Male figures are shown slim-waisted but with less heavy
abdoman as compared to those in Vijayanagara. The artist, as in the previous centuries and
following traditions, has tried to infuse movement and make the space dynamic. The painting of
Nataraja at Tiruvalanjuli is a good example.
Kerala Murals
• Kerala painters (during the period from the sixteenth to the eighteenth century) evolved a
pictorial language and technique of their own while discriminately adopting certain stylistic
elements from Nayaka and Vijayanagara schools.
• The painters evolved a language taking cues from contemporary traditions like Kathakali and
kalam ezhuthu using vibrant and luminous colours, representing human figures in three-
dimensionality.
• Most of the paintings are seen on the walls of shrines and cloister walls of temples and
some inside palaces.
• Thematically too, paintings from Kerala stand apart. Most of the narrations are based on
those episodes from Hindu mythology which were popular in Kerala. The artist seems to
have derived sources from oral traditions and local versions of the Ramayana and the
Mahabharata for painted narration.
• More than sixty sites have been found with mural paintings which include three
palaces—Dutch palace in Kochi, Krishnapuram palace in Kayamkulam and
Padmanabhapuram palace.
• Among the sites where one can see the mature phase of Kerala’s mural painting tradition
are Pundareekapuram Krishna temple, Panayanarkavu, Thirukodithanam, Triprayar Sri
Rama temple and Trissur Vadakkunathan temple.
QnA
1.What are the main features of Badami Cave paintings?.
Main features of Badami cave paintings are as follows:
• Badami cave paintings depict palace scenes, showcasing the lifestyle and royal
activities of the Chalukyan dynasty.
•The surviving painting fragment is located on the vaulted roof of the front mandapa in
Cave No. 4, known as the Vishnu Cave.
•The painting style exhibits sinuous lines, fluid forms, and a compact composition.
•The artists achieved a high level of proficiency and maturity in the sixth century CE.
•The faces of the depicted figures, such as the king and queen, display characteristics
similar to the Ajanta style, including large eye sockets, half-closed eyes, and protruding
lips.
•The artists skillfully use contours to create volume and depth, showcasing their ability to
convey three-dimensionality through simple line treatment.

2. Write an essay on Vijayanagar Paintings.

3.Describe the murals traditions of Kerala and Tamil Nadu.


Temple Architecture and Sculpture
• Most of the art and architectural remains that survive from Ancient and Medieval
India are religious in nature. That does not mean that people did not have art in
their homes at those times. Their domestic dwellings and the things in them were
mostly made from materials like wood and clay which have perished, or were
made of metal (like iron, bronze, silver and even gold) which was melted down
and reused from time to time.
• This chapter introduces us to many types of temples from India. Although we
have focused mostly on Hindu temples, at the end of the chapter you will find
some information on major Buddhist and Jain temples too.
• However, at all times, we must keep in mind that religious shrines were also
made for many local cults in villages and forest areas, but again, not being of
stone the ancient or medieval shrines in those areas have also vanished.
The basic form of the Hindu Temple
The basic form of the Hindu temple comprises the following:
• (i) a cave-like sanctum (garbhagriha literally ‘womb-house’), which, in the early
temples, was a small cubicle with a single entrance and grew into a larger
chamber in time. The garbhagriha is made to house the main icon which is itself
the focus of much ritual attention;
• (ii) the entrance to the temple which may be a portico or colonnaded hall that
incorporates space for a large number of worshippers and is known as a
mandapa;
• (iii) from the fifth century CE onwards, freestanding temples tend to have a
mountain like spire, which can take the shape of a curving shikhar in North India
and a pyramidal tower, called a vimana, in South India;
• (iv) the vahan, i.e., the mount or vehicle of the temple’s main deity along with a
standard pillar or dhvaj is placed axially before the sanctum.
The basic form of the Hindu Temple

• Two broad orders of temples in the country are known— Nagara in the north and
Dravida in the south.
• At times, the Vesar style of temples as an independent style created through the
selective mixing of the Nagara and Dravida orders is mentioned by some scholars.
Elaborate studies are available on the various sub-styles within these orders as
well. We will look into the differences in the forms further on in this chapter.
• As temples grew more complex, more surfaces were created for sculpture
through additive geometry, i.e., by adding more and more rhythmically
projecting, symmetrical walls and niches, without breaking away from the
fundamental plan of the shrine.
Sculpture, Iconography and Ornamentation
• The study of images of deities falls within a branch of art history called
‘iconography’, which consists of identification of images based on certain symbols
and mythologies associated with them. And very often, while the fundamental
myth and meaning of the deity may remain the same for centuries, its specific
usage at a spot can be a response to its local or immediate social, political or
geographical context.
• Every region and period produced its own distinct style of images with its regional
variations in iconography.
• The temple is covered with elaborate sculpture and ornament that form a
fundamental part of its conception.
• The placement of an image in a temple is carefully planned: for instance, river
goddesses (Ganga and Yamuna) are usually found at the entrance of a
garbhagriha in a Nagara temple, dvarapalas (doorkeepers) are usually found on
the gateways or gopurams of Dravida temples, similarly, mithunas (erotic images),
navagrahas (the nine auspicious planets) and yakshas are also placed at entrances
to guard them.
Sculpture, Iconography and Ornamentation

• Various forms or aspects of the main divinity are to be found on the outer walls of
the sanctum.
• The deities of directions, i.e., the ashtadikpalas face the eight key directions on
the outer walls of the sanctum and/or on the outer walls of a temple.
• Subsidiary shrines around the main temple are dedicated to the family or
incarnations of the main deity.
• Finally, various elements of ornamentation such as gavaksha, vyala/yali, kalpa-
lata, amalaka, kalasha, etc. are used in distinct ways and places in a temple.
The Nagara or North Indian Temple Style

• The style of temple architecture that became popular in northern India is known as
nagara.
• In North India it is common for an entire temple to be built on a stone platform with
steps leading up to it. Further, unlike in South India it does not usually have elaborate
boundary walls or gateways.
• While the earliest temples had just one tower, or shikhara, later temples had several.
• The garbhagriha is always located directly under the tallest tower.
• Amalaka or Kalash which is installed on Shikhara is another characteristic feature of this
form of temple style.
• Some examples of nagara style of temles - Sun temple, Konark, Sun temple at Modhera,
Gujarat and Ossian temple, Gujarat, Kandariya Mahadev Temple in Madhya Pradesh.
The Nagara or North Indian Temple Style
• There are many subdivisions of nagara temples depending on the shape of the shikhara.
• There are different names for the various parts of the temple in different parts of India;
however, the most common name for the simple shikhara which is square at the base
and whose walls curve or slope inward to a point on top is called the 'latina' or the rekha-
prasada type of shikara.
• Early medieval temples such as the Sun Temple at Markhera in Madhya Pradesh (MP).
The Sri Jagannath Temple of Odisha has been constructed in the Rekha-Prasad Shikara
style.
• Shekari is a variation of the Latina where the Shikara comprises of a main Rekha-Prasad
Shikara and one or more rows of smaller steeples on both sides of the central spire.
Additionally, the base and corners also feature mini Shikaras. The Khajuraho Kandariya
Mahadev Temple is one of the most prominent temples built in this style.
• Bhumija is another type of Nagara temple that evolved from the Latina style
that developed in Malwa under the Paramara dynasty. These temples have a flat upward
tapering projection comprising of a central Latina spire and miniature spires on the
quadrant formed by the tapering tower. These mini Shikaras carved out both horizontally
as well as vertically. The Udayeshwar Temple in MP is built in this style.
The Sri Jagannath Temple of Odisha
The Nagara or North Indian Temple Style

• The second major type of architectural form in the nagara order is the phamsana.
Phamsana are shorter but broader structures comprising of roofs with numerous slabs
that rise upwards in a gentle slope on a straight incline like a pyramid meeting at a single
point over the mid-point of the building, unlike the latina ones which look like sharply
rising tall towers. The Jagmohan of Konark Temple is constructed in the Phamsana order.
• In many North Indian temples you will notice that the phamsana design is used for the
mandapas while the main garbhagriha is housed in a latina building.
The Jagmohan of Konark Temple
The Nagara or North Indian Temple Style

• The third main sub-type of the nagara building is what is generally called the valabhi
type. These are rectangular buildings with a roof that rises into a vaulted chamber. The
edge of this vaulted chamber is rounded, like the bamboo or wooden wagons that would
have been drawn by bullocks in ancient times. They are usually called ‘wagon vaulted
buildings’.
• The form of the temple is influenced by ancient building forms that were already in
existence before the fifth century CE. The valabhi type of building was one of them. For
instance, if you study the ground-plan of many of the Buddhist rock-cut chaitya caves,
you will notice that they are shaped as long halls which end in a curved back. From the
inside, the roof of this portion also looks like a wagon-vaulted roof.
• Teli Ka Mandir, a 9th Century temple at Gwalior has been built in this style.
Teli Ka Mandir at Gwalior
Central India
• Ancient temples of Uttar Pradesh, Madhya Pradesh and Rajasthan share many traits. The
most visible is that they are made of sandstone.
• Some of the oldest surviving structural temples from the Gupta Period are in Madhya
Pradesh. These are relatively modest-looking shrines each having four pillars that
support a small mandapa which looks like a simple square porch-like extension before an
equally small room that served as the garbhagriha.
• Deogarh Temple - The patrons and donors of the temple at Deogarh (in Lalitpur District,
Uttar Pradesh) are unknown; however on the basis of both architecture and imagery, it is
established that this temple was built in the early sixth century CE. That is, about a
hundred years or so after the small temples we just learnt about. This makes it a classic
example of a late Gupta Period type of temple.
• This temple is in the panchayatana style of architecture where the main shrine is built on
a rectangular plinth with four smaller subsidiary shrines at the four corners (making it a
total number of five shrines, hence the name, panchayatana).
Central India
• The tall and curvilinear shikhara also corroborates this date. The presence of this curving
latina or rekha-prasada type of shikhara also makes it clear that this is an early example
of a classic nagara style of temple.
• This west-facing temple has a grand doorway with standing sculptures of female figures
representing the Ganga on the left side and the Yamuna on the right side.
• The temple depicts Vishnu in various forms, due to which it was assumed that the four
subsidiary shrines must also have housed Vishnu’s avatars. In fact, it is not actually
known to whom the four subsidiary shrines were originally dedicated. There are three
main reliefs of Vishnu on the temple walls: Sheshashayana on the south, Nara Narayan
on the east and Gajendramoksha on the west.
• The temple is west-facing, which is less common as most temples are east- or north-
facing.
Central India
• In contrast, if we study the temples of Khajuraho made in the tenth century, i.e., about
four hundred years after the temple at Deogarh, we can see how dramatically the shape
and style of the nagara temple architecture had developed.
• The Lakshmana temple dedicated to Vishnu is the grandest temple of Khajuraho, built in
954 by the Chandela king, Dhanga. It is a nagara temple, it is placed on a high platform
accessed by stairs. There are four smaller temples in the corners, and all the towers or
shikharas rise high, upward in a curved pyramidal fashion, emphasising the temple’s
vertical thrust ending in a horizontal fluted disc called an amalak topped with a kalash or
vase. The crowning elements: amalak and kalash, are to be found on all nagara temples
of this period.
• The temple also has projecting balconies and verandahs.
• Khajuraho’s temples are also known for their extensive erotic sculptures; the erotic
expression is given equal importance in human experience as spiritual pursuit, and it is
seen as part of a larger cosmic whole. Many Hindu temples therefore feature mithun
(embracing couple) sculptures, considered auspicious. Usually, they are placed at the
entrance of the temple or on an exterior wall or they may also be placed on the walls
between the mandapa and the main shrine.
Central India
• Khajuraho’s sculptures are highly stylised with typical features: they are in almost full
relief, cut away from the surrounding stone, with sharp noses, prominent chins, long
slanting eyes and eyebrows.
• The other notable example at Khajuraho is Kandariya Mahadeo temple dedicated to
Lord Shiva.
• At Khajuraho, There are some Jain temples as well as a Chausanth Yogini temple. This is
a temple of small, square shrines of roughly-hewn granite blocks, each dedicated to
esoteric devis or goddesses associated with the rise of Tantric worship during the 7th
century. Several such temples were dedicated to the cult of the yoginis across Madhya
Pradesh, Odisha and even as far south as Tamil Nadu. They were built between the
seventh and tenth centuries, but few have survived.
West India

• Temples in the north-western parts of India, including Gujarat, Rajasthan, and western
Madhya Pradesh, are numerous and exhibit a range of architectural styles.
• The temples are constructed using different types and colors of stones. While sandstone
is common, basalt can be found in some 10th-12th century temple sculptures. Soft white
marble is also used in Jain temples in Mount Abu and the 15th century temple at
Ranakpur.
• Samlaji in Gujarat is an important art-historical site that showcases a distinct style of
sculpture resulting from the fusion of earlier artistic traditions with a post-Gupta style.
• Grey schist sculptures, dating between the 6th and 8th centuries CE, have been
discovered in the region. The exact patrons are debated, but the dating is based on
stylistic analysis.
West India

• The Sun temple at Modhera dates back to early eleventh century and was built by Raja
Bhimdev I of the Solanki Dynasty in 1026. The Solankis were a branch of the later
Chalukyas. There is a massive rectangular stepped tank called the surya kund in front of
it. Proximity of sacred architecture to a water body such as a tank, a river or a pond has
been noticed right from the earliest times. This hundred-square-metre rectangular pond
is perhaps the grandest temple tank in India.
• A hundred and eight miniature shrines are carved in between the steps inside the tank. A
huge ornamental arch-torana leads one to the sabha mandapa (the assembly hall) which
is open on all sides, as was the fashion of the times in western and central Indian
temples.
• The influence of the woodcarving tradition of Gujarat is evident in the lavish carving and
sculpture work. However, the walls of the central small shrine are devoid of carving and
are left plain as the temple faces the east and, every year, at the time of the equinoxes,
the sun shines directly into this central shrine.
East India

• Eastern Indian temples include those found in the North East, Bengal and Odisha.
• Each of these three areas produced distinct types of temples.
• The history of architecture in the North-East and Bengal is hard to study because a
number of ancient buildings in those regions were renovated, and what survives now
are later brick or concrete temples at those sites.
• It appears that terracotta was the main medium of construction, and also for moulding
plaques which depicted Buddhist and Hindu deities in Bengal until the seventh century.
• A large number of sculptures have been found in Assam and Bengal which shows the
development of important regional schools in those regions.
Assam
• An old 6th century sculpted door frame from DaParvatia near Tezpur and another
few stray sculptures from Rangagora Tea Estate near in Assam bear witness to
the import of the Gupta idiom in that region.
• This post-Gupta style continued in the region well into the 10th century.
However, by the 12th to 14th centuries, a distinct regional style developed in
Assam.
• The style that came with the migration of the Tais from Upper Burma mixed with
the dominant Pala style of Bengal and led to the creation of nilachal style.
• Nilachal is a style of Hindu temple architecture in Assam, that is characterized by
a bulbous polygonal dome over a cruciform ratha type bada.
• This hybrid style developed first in the Kamakhya temple, a Shakti Peeth built in
the 17th century on the Nilachal hills under the Koch kingdom and became
popular later under the Ahom kingdom.
Kamakhya temple, Sibsagar, Assam
Bengal
•Style between the 9th and 11th centuries – Pala Style. The Palas were patrons of
Buddhist monastic styles. The temples in this region showcased the local Vanga style.
•Style of temple architecture from the middle of the 11th century to the middle of the
13th centuries – Sena style.
•Siddheswara Mahadeva temple in Barakar in Burdwan District – 9th century; tall curving
Shikhara crowned by a large amalaka is an example of early Pala style.
•Many temples were located at Telkupi in Purulia District – 9th to 12th century but were
submerged due to dam construction. These temples showed all the Nagara sub-styles
prevalent in the north.
•Some temples survive.
•Made of black to grey basalt.
•Had chlorite stone pillars and arched niches.
•They influenced early Bengal Sultanate buildings at Gaur and Pandua.
Siddheshvara Mahadeva temple, Barakar, Burdwan District.
Bengal
•Local vernacular building traditions also influenced the temples. Most noticeable of these
influences was the curving or sloping side of the bamboo roof of a Bengali hut.
•This feature was adopted in Mughal buildings and is known as the Bangla Roof.
•From the Mughal period onwards, several terracotta brick temples were built which had
elements from the earlier Pala style, from the local bamboo hut styles, and arches and
domes from Islamic architecture.
•Eg. Terracotta Temple, Vishnupur (17th century).

Bangla Roof
Bengal
•The main architectural features of Odisha temples are classified in three orders, i.e.,
rekhapida, pidhadeul and khakra.
•Most of the main temple sites are located in ancient Kalinga—modern Puri District,
including Bhubaneswar or ancient Tribhuvanesvara, Puri and Konark.
•The temples of Odisha constitute a distinct sub-style within the nagara order.
•In general, here the shikhara, called deul in Odisha, is vertical almost until the top when it
suddenly curves sharply inwards.
•Deuls are preceded, as usual, by mandapas called jagamohana in Odisha.
•The ground plan of the main temple is almost always square, which, in the upper reaches
of its superstructure becomes circular in the crowning mastaka. This makes the spire
nearly cylindrical in appearance in its length.
•Compartments and niches are generally square, the exterior of the temples are lavishly
carved, their interiors generally quite bare.
Khakra duel
Bengal
•Odisha temples usually have boundary walls.
•At Konark, on the shores of the Bay of Bengal, lie the majestic ruins of the Sun temple
built in stone around 1240. Its duel was a colossal creation said to have reached 70m,
which, proving too heavy for its site, fell in the 19th century.
•The vast complex is within a quadrilateral precinct of which the jagamohana or the
dance-pavillion (mandapa) has survived, which is no longer accessible is said to be the
largest enclosed space in Hindu architecture.
•The Sun temple is set on a high base, its walls covered in extensive, detailed ornamental
carving. These include twelve pairs of enormous wheels sculpted with spokes and hubs,
representing the chariot wheels of the Sun god who, in mythology, rides a chariot driven
by eight horses, sculpted here at the entrance staircase.
Bengal
•The whole temple thus comes to resemble a colossal processional chariot. On the
southern wall is a massive sculpture of surya carved out of green stone. It is said that there
were three such images, each carved out of a different stone placed on the three temple
walls, each facing different directions. The fourth wall had the doorway into the temple
from where the actual rays of the sun would enter the garbhagriha.

Konark Sun Temple Complex illustrated


Hills
• A unique form of architecture developed in the hills of Kumaon, Garhwal, Himachal and
Kashmir.
• Kashmir’s proximity to prominent Gandhara sites (such as Taxila, Peshawar and the
northwest frontier) lent the region a strong Gandhara influence by the 5th century CE.
• This began to mix with the Gupta and post-Gupta traditions that were brought to it
from Sarnath, Mathura and even centres in Gujarat and Bengal.
• Brahmin pundits and Buddhist monks frequently travelled between Kashmir, Garhwal,
Kumaon and religious centres in the plains like Banaras, Nalanda and even as far south
as Kanchipuram. As a result both Buddhist and Hindu traditions began to intermingle
and spread in the hills.
• The hills also had their own tradition of wooden buildings with pitched roofs. At several
places in the hills, therefore, you will find that while the main garbhagriha and shikhara
are made in a rekha-prasada or latina style, the mandapa is of an older form of wooden
architecture.
Hills
• Sometimes, the temple itself takes on a pagoda shape.
• The Karkota period of Kashmir is the most significant in terms of architecture.
• One of the most important temples is Pandrethan, built during the 8th and 9th
centuries. In keeping with the tradition of a water tank attached to the shrine, this
temple is built on a plinth built in the middle of a tank.
• Although there are evidences of both Hindu and Buddhist followings in Kashmir,
this temple is a Hindu one, possibly dedicated to Shiva.
• The architecture of this temple is in keeping with the age-old Kashmiri tradition
of wooden buildings. Due to the snowy conditions in Kashmir, the roof is peaked
and slants slowly outward.
• The temple is moderately ornamented, moving away from the post-Gupta
aesthetics of heavy carving. A row of elephants at the base and a decorated
doorway are the only embellishments on the shrine.
Pandrethan Temple
Hills
• Like the findings at Samlaji, the sculptures at Chamba also show an
amalgamation of local traditions with a post-Gupta style.
• The images of Laksna-Devi Mandir are evidences of the influence of the post-
Gupta tradition. This temple bears an inscription that states that it was built
during the reign of Meruvarman who lived in the 7th century.

Lakshmi Narayana Temple, Chamba Laksna Devi Mandir

You might also like